微觀經濟學

最低要求4444不同的商品

  • May 19, 2021

消費者的效用函式為 $ 4 $ 商品

$$ U=\min\left { \sqrt{x+y},z+w \right } $$價格是 $ p=(3,2,2,1) $ 有工資 $ m=1 $ .

  • 找到需求。

到目前為止,我觀察到它是完美的補充,但它必須介於 4 種商品之間。所以一些組合 $ x,y $ 到 $ z,w $ . 通常我們知道最優選擇是當 $ x_2=x_1 $ 有需求 $ x_{1,2}=m/(p_1+p_2) $ (但是有 4 件商品,我失去了控制權。我該如何處理?我是否只是將右側平方並讓兩者相等?

您可以通過注意效用函式的嵌套結構順序解決它 $ U $ .

所以首先請注意,效用函式結合了您可能已經熟悉的效用函式 $ U=\min{u_1,u_2} $ 的補充和 $ u_1=\sqrt{x+y} $ 和 $ u_2 = z+w $ 兩者都是完美的替代品。在哪裡 $ u_1 $ 和 $ u_2 $ 嵌套在 $ U $ .

解決過程基於以下論點:無論多少錢 $ m_{u_1} $ 代理決定花費 $ u_1 =\sqrt{x+y} $ 她必須盡量減少獲得的成本 $ u_1 $ 實用點。同樣,無論多少錢 $ m_{u_2} $ 代理花費在 $ u_2= z + w $ 必須是代理最小化成本的情況。所以首先找到這些內部問題的支出函式。

其次,代理人必須最大化外部效用 $ \min{u_1,u_2} $ 以內部問題的支出加總收入為準。

解決第二個內部問題:

第二個內在問題是

[數學處理錯誤]$$ max_{z,w} {z+w \lvert p_zz+p_ww\leq m_{u_2}}, $$ 在哪裡[mu2Math Processing Error]錢花在[u2Math Processing Error]. $ m_{u_2} $ $ u_2 $

當實用程序是實用程序時,您如何獲得最大的實用程序 $ u_2 = z + w $ ? 你只要買最便宜的東西那麼一分的價格[Math Processing Error]效用是[Math Processing Error]. 的需求[Math Processing Error]和[Math Processing Error]取決於你決定花多少錢[Math Processing Error]假設現在是[Math Processing Error]. 鑑於這一數量的需求[Math Processing Error]是 $ u_2 $ $ p_{u_2} := \min{p_z,p_w} $ $ z $ $ w $ $ u_2 $ $ m_{u_2} $ $ z $

[數學處理錯誤]$$ z = \frac{m_{u_2}}{p_z} 1[p_z <p_w] $$ 和

[數學處理錯誤]$$ w = \frac{m_{u_2}}{p_w} 1[p_w <p_z]. $$ 我把它留給你來處理門檻值情況 $ p_z=p_w $ . 多少效用[Math Processing Error]你明白了嗎?嗯,這一定是[Math Processing Error]- 這是這個內部效用最大化問題的價值函式 - 因此它必須是這樣的情況 $ u_2 $ $ u_2 = m_{u_2}/\min{p_z,p_w} $

[數學處理錯誤]$$ m_{u_2} = \min{p_z,p_w} u_2, $$ 這是這個內部問題的支出函式。

解決第一個內部問題:

第一個內在問題是

[數學處理錯誤]$$ max_{x,y} {\sqrt{x+y} \lvert p_xx+p_yy\leq m_{u_1}}, $$ 在哪裡[Math Processing Error]錢花在[Math Processing Error]. $ m_{u_1} $ $ u_1 $

當實用程序是實用程序時,您如何獲得最大的實用程序 $ u_1 = \sqrt{x+y} $ ? 它是完美的替代品,因此您購買最便宜的商品,因此需求量很大

$$ x = \frac{m_{u_1}}{p_x} 1[p_x <p_y] $$ 和

$$ y = \frac{m_{u_1}}{p_y} 1[p_y <p_x], $$ 在哪裡[Math Processing Error]是花費的金額[Math Processing Error]假定已知。多少效用 $ m_{u_1} $ $ u_1 $ $ u_1 $ 你明白了嗎?那一定是 $ u_1 = \sqrt{m_{u_1}/\min{p_x,p_y}} $ 這樣

$$ m_{u_1} = \min{p_x,p_y} u_1^2 $$ 解決外部問題:

外部問題是

[數學處理錯誤]$$ \max_{u_1,u_2}: \ \ \ \min{u_1,u_2} \[8pt] s.t. \ \ \ \ m_{u_1} + m_{u_2} = m $$ 我們將預算約束重寫為

$$ \min{p_x,p_y} u_1^2 + \min{p_z,p_w} u_2 = m $$ 然後使用它 $ u_1=u_2 $ 因為問題是完美的補充。然後你有兩個未知數的兩個方程 $ u_1 $ 和 $ u_2 $ .求解 $ u_1 $ 和 $ u_2 $ 並將解決方案重新插入支出函式中,以獲得內部問題 $ m_{u_1} $ 和 $ m_{u_2} $ 然後將其代入需求方程。

引用自:https://economics.stackexchange.com/questions/44046